Thomas' Calculus 13th Edition

Published by Pearson
ISBN 10: 0-32187-896-5
ISBN 13: 978-0-32187-896-0

Chapter 5: Integrals - Section 5.2 - Sigma Notation and Limits of Finite Sums - Exercises 5.2 - Page 266: 42

Answer

$1$

Work Step by Step

Consider $f(x)=3x^2$ Here,we have $\Sigma_{i=1}^n (\dfrac{3}{n}) (k_i)^2=(\dfrac{3}{n^3}) \Sigma_{i=1}^n (i^2)$ This implies that $(\dfrac{3}{n^3})(\dfrac{n(n+1)(2n+1)}{6})=\dfrac{2n^3+3n^2+n}{2n^3}=\dfrac{2+\dfrac{3}{n}+\dfrac{1}{n^2}}{2}$ Now, $\lim\limits_{n \to \infty}\dfrac{2+\dfrac{3}{n}+\dfrac{1}{n^2}}{2}=1$
Update this answer!

You can help us out by revising, improving and updating this answer.

Update this answer

After you claim an answer you’ll have 24 hours to send in a draft. An editor will review the submission and either publish your submission or provide feedback.